LESSON Sine and Cosine Ratios 13-2 Practice and Problem Solving: A/B After verifying that the triangle to the right is a right triangle, use a calculator to find the given measures. Give ratios to the nearest hundredth and angles to the nearest degree. 1. Use the Pythagorean Theorem to confirm that the triangle is a right triangle. Show your work.

Answers

Answer 1

I saw this

is this correct? to start solving your question?

1.- Pythagorean theorem

6^2 = (5.2)^2 + (3)^2

36 = 27.04 + 9

36 = 36 Yes, it is a right triangle

2.-

[tex]\begin{gathered} \sin \text{ 1 = }\frac{5.2}{6} \\ \sin 2\text{ = }\frac{3}{6}\text{ = }\frac{1}{2} \end{gathered}[/tex]

LESSON Sine And Cosine Ratios 13-2 Practice And Problem Solving: A/B After Verifying That The Triangle

Related Questions

Ainsley had 3 yards of string. She used 1/4 yard for a project. She wants to divide the rest into pieces 5/6 yard long. How many pieces can she make?

Answers

She can make 3.3 pieces from the remaining length

How to determine the number of pieces to make?

From the question, the given parameters are:

Length of string = 3 yardsAmount used for project = 1/4 yardLength of each piece = 5/6 yard

We start by calculating the length of the remaining pieces as a whole

This is calculated as

Length of remaining piece + length used for project = Length of string

Substitute the known values in the above equation

So, we have

Length of remaining piece + 1/4 yard = 3 yards

Subtract 1/4 from both sides

Length of remaining piece = 2.75 yards

The number of pieces is then calculated as

Pieces = Length of remaining pieces/Length of each piece

This gives

Pieces = 2.75 yards/(5/6 yards)

Evaluate the quotient

Pieces = 3.3

Hence, the number of pieces she can make is 3.3

Read more about lengths at

https://brainly.com/question/24487155

#SPJ1

17) A brand of coconut oil is updating its packaging. The old rectangular container was 1 foottall, 4 inches long and 2.5 inches wide, their new cylindrical container has a 4.5 centimeterradius and is 30 centimeters tall.a) Determine the volume of the rectangular container, 3 points(if needed round the final answer to the nearest hundredth.)b) Determine the volume of the cylindrical container. 3 points(if needed round the final answer to the nearest hundredth.)c) The company charges the same amount for each container. Is one of the containers abetter buy? Explain and show work that justifies your answer. 3 points

Answers

a) Volume of rectangular container:

1 foot = 12 inches

[tex]\begin{gathered} V=\text{tall}\times long\times wide \\ V=12\times4\times2.5 \\ V=120inches^3 \end{gathered}[/tex]

b) Volume of the cylindrical container:

[tex]\begin{gathered} V=\pi\times r^2\times h \\ V=\pi\times(4.5)^2\times30 \\ V=\pi\times20.25\times30 \\ V=607.5\pi \\ V=607.5(3.14) \\ V=1907.55\operatorname{cm}^3 \end{gathered}[/tex]

c) First, we convert the volumes of the containers into the same units.

For volume of the rectangular container in centimeters:

1 inch3 = 16.3871 cm3

[tex]V=120\times16.3871=1966.45\operatorname{cm}^3[/tex]

With this, we can say that the rectangular container has a larger volume than the cylindrical container. Therefore, the cylindrical container is a better buy.

Answer:

a) V = 120 in^3

b) V = 1907.55 cm^3

c) Cylindrical container is a better buy

Solve g(7) given g(X)= -3x+1

Answers

Step-by-step explanation:

that means we need to calculate the function result with x having a specific value.

for this we replace any appearance of x in the function expression by that specific value, and then we calculate.

g(x) = -3x + 1

g(7) means x = 7, and that gives us

-3×7 + 1 = -21 + 1 = -20

so, g(7) = -20

Answer:

-20

Step-by-step explanation:

Substitute 7 into the function g(x)

Since negatives "rule" positives,

7 * -3 = -21,

-21 + 1 = -20

Mrs. Ojo is 5 times older than her son. 3 years ago the product of their ages was 185. How old are they now?​
need it now please.

Answers

A= 5B •••> (1)
(A – 3)(B – 3)= 185
AB – 3A – 3B + 9= 185
AB – 3(A + B)= 185 – 9= 176
AB – 3(A + B)= 176 •••> (2)

Put (1) in (2),
5B(B) – 3(5B + B)= 176
5B² – 3(6B)= 176
5B² – 18B – 176= 0
(B – 8)(B + 22/5)= 0
B= 8 and B= –22/5

When B= 8, from (1),
A= 5(8)= 40.

Thus, Mrs Ojo is 40 while her son is 8.

given the preimage coordinates complete the translation provide and give the image Coordinates

Answers

The coordinates are made up of an x-​​coordinate and a y-coordinate, written in the form (x, y), the translation given is (x + 1, y - 1), which means adding +1 to all x-coordinates and subtracting -1 to all y-coordinate.

Then, the pair B(0, -4) will be transformed into B'(0 + 1, -4 - 1) → B'( 1, -5)

C( -1, 1) → C' (-1 + 1, 1 - 1) → C'( 0, 0)

D( 1, 3) → D' ( 1 + 1, 3 - 1) → D'( 2, 2)

E( 4, -1) → E' ( 4 + 1, -1 - 1) → E'( 5, -2)

Emery borrowed money from her brother to buy a new phone and is paying off a fixed amount each week. After 2​ weeks, she will owe​ $456, and after 5​ weeks, she will owe​ $228.
a. What was the original amount Emery​ borrowed?
b. How much does she pay each​ week?
c. How useful are equations in​ point-slope and​ slope-intercept forms for answering each​ question?

Answers

The $456 and $228 Emery owed from the amount she borrowed to buy a new phone after 2 weeks and 5 weeks gives;

a. The original amount owed is $608

b. Emery pays $76 each week

c. The point and slope and slope and intercept both gives the amount Emery pays each week as the slope, while the slope and intercept form further gives the original amount borrowed as the y–intercept, c

What is a straight line equation?

A straight line equation is one which expresses a linear relationship between variables

Let y represent the original amount of money Emery borrowed from her brother, and let m represent the amount she pays each week to settle the loan, we have;

The given parameters are;

The amount Emery owes after 2 weeks = $456

The amount Emery will owe after 5 weeks = $228

a. The above information can be expressed by writing the following equations;

456 = y - 2•m...(1)

228 = y - 5•m...(2)

Subtracting equation (2) from equation (1) gives;

y - 2•m - (y - 5•m) = 456 - 228 = 228

3•m = 228

Therefore;

m = 228 ÷ 3 = 76

m = 76

The original amount Emery borrowed, y, from equation (1) is therefore;

y = 456 + 2•m

y = 456 + 2 × 76 = 608

The original amount Emery borrowed, y, is $608

b. The amount Emery pays each week, m = $76

c. The straight line equation in point and slope form is presented as follows;

[tex] y_2 - y_1 = m•(x_2 - x_1) [/tex]

From the above equation, the two points (456, 2) and (228, 5) given in the question can be used to find the slope, m, which is the amount paid each month

The slope and intercept form of a straight line equation is y = m•x + c

From the slope and intercept form, we have;

The slope, m = The amount Emery pays per month

The intercept, c = The original amount Emery borrowed

Therefore, both forms of the straight line equation gives the amount Emery pays each week, while the slope and intercept form also gives the original amount Emery borrowed

Learn more about the equations of a straight line here:

https://brainly.com/question/28778169

#SPJ1

Determine the odds against rolling a 4 on one roll
of a die.
Solution
P(4)=1
6
5
P (fails to roll a 4)=-
6

Answers

Answer:

1/6

Step-by-step explanation:

SIX possible rolls    1,2,3,4,5,6

   4 is 1 out of 6      or 1/6

The equation of a curve is xy squared minus 2x square y squared equal 0. Find the gradient of the tangent to the curve at (1,2)?

Answers

Solution

[tex]xy^2-2x^2y^2=0[/tex]

we need to find the derivative with respect to x. dy/dx

[tex]\begin{gathered} \frac{d}{dx}(xy^2-2x^2y^2=0)=\frac{d}{dx}(xy^2)-2\frac{d}{dx}(x^2y^2)=0 \\ \\ \text{ using product rule} \\ \\ \Rightarrow x\frac{d}{dx}(y^2)+y^2\frac{d}{dx}(x)-2x^2\frac{d}{dx}(y^2)+2y^2\frac{d}{dx}(x^2)=0 \\ \end{gathered}[/tex]

Applying chain rule

[tex]\begin{gathered} x\cdot\frac{d}{dy}(y^2)\cdot\frac{dy}{dx}+y^2-2x^2\cdot\frac{d}{dy}(y^2)\cdot\frac{dy}{dx}+2y^2(2x)=0 \\ \\ \Rightarrow x(2y)\frac{dy}{dx}+y^2-2x^2(2y)\frac{dy}{dx}+4xy^2=0 \\ \\ \Rightarrow2xy\frac{dy}{dx}+y^2-4x^2y\frac{dy}{dx}+4xy^2=0 \\ \\ \Rightarrow(2xy-4x^2y)\frac{dy}{dx}=-y^2-4xy^2 \\ \\ \Rightarrow\frac{dy}{dx}=\frac{-(y^2+4xy^2)}{2xy-4x^2y}=\frac{y^2+4xy^2}{4x^2y-2xy} \\ \\ \Rightarrow\frac{dy}{dx}=\frac{y^{2}+4xy^{2}}{4x^{2}y-2xy} \end{gathered}[/tex]

At point (1,2)

[tex]\frac{dy}{dx}=\frac{(2)^2+4(1)(2)^2}{4(1)^2(2)-2(1)(2)}=5[/tex]

Using slope intercept equation

[tex]\begin{gathered} y-y_1=m(x-x_1) \\ \\ y-2=5(x-1) \\ \\ y-2=5x-5 \\ \\ \Rightarrow y=5x-5+2 \\ \\ \Rightarrow y=5x-3 \end{gathered}[/tex]

question shown in picture below!!(i only need the 2nd one)

Answers

Given the following sequence:

[tex]\mleft\lbrace17,14,11,8,\ldots\mright\rbrace[/tex]

notice that the common difference between each term is -3, therefore, we can use the explicit formula for a sequence:

[tex]\begin{gathered} a_n=a_1+(n-1)\cdot d \\ d=-3 \\ a_1=17 \\ \Rightarrow a_n=17+(n-1)(-3) \\ \Rightarrow a_n=17-3n+3=20-3n \\ a_n=20-3n \end{gathered}[/tex]

we have that the explicit formula is a_n=20-3n, while the recursive formula is:

[tex]\begin{gathered} a_n=a_{n-1}+d \\ d=-3 \\ \Rightarrow a_n=a_{n-1}-3 \end{gathered}[/tex]

Select the vector along which a translation of the plane would map point A to its image T(A).

Answers

The vector along which a translation of the plane would map point A to its image T(A) is: C. vector MN.

The types of transformation.

In Geometry, there are different types of transformation and these include the following:

ReflectionDilationRotationTranslation

What is a translation?

A translation can be defined as a type of transformation which moves every point of the object in the same direction, as well as for the same distance.

In this context, we can reasonably infer and logically deduce that vector MN is a translation of the given plane which would map point A to its image point T(A) because they both move in the same direction.

Read more on translation here: https://brainly.com/question/18921791

#SPJ1

The ratio of two cars, the Thunderbolt and the Flash, in terms of their miles per gallon was 6:5. The new version of the Flash added 14 miles per gallon and now the ratio is 24:27 .

​What is the miles per gallon of the Thunderbolt?



​The Thunderbolt gets
miles per gallon.

Answers

If the ratio the Thunderbolt and the Flash, in terms of their miles per gallon was 6:5 and the new version of the Flash added 14 miles per gallon and now the ratio is 24:27, then the miles per gallon of the Thunderbolt is  15.27 miles per gallon

The ratio the Thunderbolt and the Flash, in terms of their miles per gallon = 6:5

Then the miles per gallon of the Thunderbolt and the Flash is 6x and 5x respectively

The new version of the Flash added 14 miles per gallon

New ratio of the cars = 24:27

Therefore the equation is

5x+14 = 27x

27x-5x = 14

22x = 14

x = 7/11

The miles per gallon of the Thunderbolt = 24x

= 24×7/11

= 15.27 miles per gallon

Hence, If the ratio the Thunderbolt and the Flash, in terms of their miles per gallon was 6:5 and the new version of the Flash added 14 miles per gallon and now the ratio is 24:27, then the miles per gallon of the Thunderbolt is 15.27 miles per gallon

Learn more about ratio here

brainly.com/question/13419413

#SPJ1

How do I solve this problem? To find the distance AB across a river, a distance BC=250 is laid off on one side of the river. It is found that B =119 degrees and C=27 degrees. Find AB

Answers

[tex]\begin{gathered} \text{To find A} \\ A+119+27=180 \\ A=180-119-27 \\ A=34 \\ U\sin g\text{ sine theorem} \\ \frac{AB}{\sin(27)}=\frac{BC}{\sin(34)} \\ \text{Solving AB} \\ AB=\frac{BC\sin(27)}{\sin(34)} \\ BC=250 \\ AB=\frac{250\sin(27)}{\sin(34)} \\ AB=202.966\approx203 \\ \text{The distance AB is 203} \\ \\ \text{Question 2} \\ To\text{ find the }height\text{ of the h}ill \\ height\text{ = OB} \\ \tan (10)=\frac{OB}{600} \\ \text{Solving OB} \\ OB=600\tan (10) \\ OB=105.796\text{ fe}et\approx106\text{ fe}et \\ \text{The height of the hill is 106 ft} \\ To\text{ find the height of the antenna} \\ \text{Antenna}=OT-OB \\ \tan (25)=\frac{OT}{600} \\ \text{Solving OT} \\ OT=600\tan (25) \\ OT=279.78\text{feet}\approx279.8feet \\ \text{Hence} \\ \text{Antenna}=279.8feet-106\text{ fe}et \\ \text{Antenna}=137.8\text{ fe}et \\ \text{The height of the antenna is 137.8fe}et \end{gathered}[/tex]

1. Fiber-Brite could clean 48 rugs in 12 hours. At that rate, how many rugs could Fiber-Brite
clean in 20 hours?

Answers

80 rugs cleaned in 20 hours

There are about 3.79 liters in 1 gallon. Kendra’s fish tank contains 100 gallons of salt water. About how many liters does the tank hold? Round your answer to the nearest whole number. Answer to get 60 points plus brainliest!

Answers

Answer:

379 liters

Step-by-step explanation:

3.79  L / gal   * 100 gal = 379 Liters

Kendra's fish tank can hold 379 liters of water under the given conditions.

Given that,
There are about 3.79 liters in 1 gallon. Kendra’s fish tank contains 100 gallons of salt water.

What is simplification?

The process in mathematics to operate and interpret the function to make the function or expression simple or more understandable is called simplifying and the process is called simplification.

Here,
3.79 liters = 1 gallon,
Now,
in the fish tank
100 gallon of water = 3.79× 100 liters
                                 = 379 liters

Thus, Kendra's fish tank can hold 379 liters of water under the given conditions.

Learn more about simplification here:

https://brainly.com/question/12501526

#SPJ2

if the sum of the first three terms of a GP is half it's sum to infinity. find it's positive common ratio.​

Answers

Answer:

See below

Step-by-step explanation:

Sum of n = inf    =     a / ( 1-r)       one half of this = 1/2 ( a/(1-r) )

Sum of first 3 terms =   a ( 1- r^n) / ( 1-r)   = a (1-r^3) / (1-r)

The underlined value are equal ( given in the Q)

1/2  a / (1-r)    = a ( 1-r^3) / (1-r)           Multiply both side by ( (1-r) )

1/2 a = a ( 1-r^3)           divide both sides by  a

1/2 = 1- r^3      subtract 1 from both sides

-1/2 = - r^3

1/2 = r ^3

r = cuberoot ( 1/2)  =  1/2 ^(1/3)  =   .793700526

S is the midpoint of segment OU. OS= 7x-9 and SU=4x+3. Determine the length of segment OU.

Answers

Answer:

The equation for the value of OU is;

[tex]OU=11x-6[/tex]

Explanation:

Given that S is the midpoint of segment OU.

Then;

[tex]OU=OS+SU[/tex]

Given in the question;

[tex]\begin{gathered} OS=7x-9 \\ SU=4x+3 \end{gathered}[/tex]

Substituting the values of OS and SU to get OU;

[tex]\begin{gathered} OU=OS+SU \\ OU=7x-9+4x+3 \\ OU=7x+4x-9+3 \\ OU=11x-6 \end{gathered}[/tex]

Therefore, the equation for the value of OU is;

[tex]OU=11x-6[/tex]

how do I find the given z score

Answers

The formula for calculating a z-score is

[tex]z=\frac{x-\mu}{\sigma}[/tex]

Where:

*x is the raw score

*μ is the population mean

*σ is the population standard deviation

in a 30 16 90° triangle given the short leg equals five find the long leg of the triangle

Answers

In a 30-60-90 triangle:

[tex]\begin{gathered} Hypotenuse_{\text{ }}=_{\text{ }}2\cdot short_{\text{ }}length \\ Long_{\text{ }}length=\sqrt[]{3}\cdot short_{\text{ }}length \\ \end{gathered}[/tex]

so:

[tex]\begin{gathered} Long_{\text{ }}length=\sqrt[]{3}\cdot5 \\ Long_{\text{ }}length=5\cdot\sqrt[]{3}\approx8.66 \end{gathered}[/tex]

Assuming the pattern continues, what is S12 for the series −5 − 14 − 23 − 32 − …?A)-104B)-624C)-654D)-663

Answers

The sum of the first 12 terms is s(12) = -624 if the sequence is series −5 − 14 − 23 − 32 option (B) is correct.

What is a sequence?

It is defined as the systematic way of representing the data that follows a certain rule of arithmetic.

It is given that:

The sequence is:

−5 − 14 − 23 − 32 −

s(12) means the sum of the 12 terms

s(12) = (12/2)[-5 + (12-1)(-9)]

s(12) = 6[-5 -99]

s(12) = 6[-104]

s(12) = -624

Thus, the sum of the first 12 terms is s(12) = -624 if the sequence is series −5 − 14 − 23 − 32 option (B) is correct.

Learn more about the sequence here:

brainly.com/question/21961097

#SPJ1

Z divided 12 equal 9

Answers

Answer:

108

Step-by-step explanation:

12×9=108

just for the extra characters

Answer:

108

Step-by-step explanation:

z/12 = 9

z = 9 × 12

z = 108

Please rate BRAINLIEST.

Factor completely x^2+10x+21

Answers

Answer:

(x+3)(x+7)

Step-by-step explanation:

x2+10x+21

The middle number is 10 and the last number is 21.

Factoring means we want something like

(x+_)(x+_)

We need two numbers that

Add together to get 10

Multiply together to get 21

3 and 7:

3+7 = 10

3*7 = 21

Fill in the blanks in

(x+_)(x+_)

with 3 and 7 to get...

(x+3)(x+7)

Answer: (x+7)(x+3)

Step-by-step explanation:

We need to values that we will add and get 10 and two values we will multiply and get 21 which are 7 and 3

A bag of cookies weighs 19.1 ounces. The bag contains 50 cookies. How much does each cookie weigh?
O2.6 ounces
O 0.3 ounces
O 0.382 ounces
O 3.82 ounces

Answers

Answer: 0.382 ounces

Step-by-step explanation:

To find the unit rate of the weight of a single cookie, we will use division.

        19.1 ounces per bag / 50 cookies in the bag = 0.382 ounces per cookie

To check our work, we will multiply 0.382 ounces per cookie by 50. Since the bag, with 50 cookies, weighs 19.1 ounces, 0.382 times 50 should equal about 19.1 ounces.

        0.382 ounces * 50 cookies = 19.1 ounces ✓

Using P.E.M.D.A.S, how do I solve this?

Answers

Answer: 7

Step-by-step explanation:

First, do parenthesis- 2x4 is 8 and 8 times - is -8. Now it is 30/6+10-8. 30 divided by 6 is 5. 10-8 is 2. 5+2 is 7.

Which of the following lists of data has the largest standard deviation? Hint: you should not need to compute the standard deviation for each list.Select the correct answer below:22, 25, 25, 22, 25, 23, 24, 23, 26, 2413, 11, 11, 11, 10, 14, 14, 10, 11, 1116, 14, 15, 15, 16, 16, 15, 16, 18, 1425, 23, 25, 22, 21, 25, 22, 21, 25, 237, 14, 19, 18, 4, 16, 5, 11, 11, 12

Answers

Answer:

7, 14, 19, 18, 4, 16, 5, 11, 11, 12

Explanation:

The standard deviation is a statistic that measures the spread of a dataset relative to its mean.

From the given options, find the range in each of them:

[tex]Range=Highest\;Value-Lowest\;Value[/tex]

• Option 1: Range = 26-22=4

,

• Option 2: Range=14-10=4

,

• Option 3: Range=18-14=4

,

• Option 4: Range = 25-21=4

,

• Option 5: Range = 19-4=15

We see that the dataset in Option 5 has a far greater range and thus the dataset will have the biggest spread relative to its mean.

Option 5 has the largest standard deviation.

I have to send you the question

Answers

The given expression fro the sum of n terms:

[tex]\begin{gathered} S_n=\frac{n}{2}(a_1+a_n) \\ \text{ Where: }a_1\text{ is the firm term \& }a_n\text{ is the last term} \end{gathered}[/tex]

From the given question we have:

[tex]a_1=8,a_n=79,\text{ n =6}[/tex]

Substitute these value in the expression of sum of n terms

[tex]\begin{gathered} S_n=\frac{n}{2}(a_1+a_n) \\ S_6=\frac{6}{2}(8+79) \\ S_6=3(87) \\ S_6=3\times87 \\ S_6=261 \end{gathered}[/tex]

So, sum of 6 terms is 261

Write TWO facts about Pi

Answers

Solution:

Two facts about pi π

1) It is an irrational number.

2) It is ancient.

4 thousands +12 hundreds =

Answers

Answer:

that's too easy

4 thousands=4000

12 hundreds=120

4000+120=4120

4 thousands and 12 hundreds

4000
4000+
4000+120
4000+120=
Answer is 4,120

(4,000+120=4,120)

help me pleaseeeeeeeeeeeeeeeeeeeeeeeee !!!

Answers

Answer:

[tex]f(x)=\frac{5}{6}x+21[/tex]

Step-by-step explanation:

Slope = 5/6

Point = (-18,6)

Using Point-Slope form

y-6=5/6(x+18)

y-6=5/6x+15

y=5/6x+21

:]

A native wolf species has been reintroduced into a national forest. Originally 200 wolves were transplanted. After 3 years, the population had grown to 270 wolves. If the population grows exponentially, then at what annual rate is the population growing? Round the answer to the nearest tenth of a percent.

Answers

The population is growing at an annual rate of 16.2%

Here, we want to calculate the exponential growth rate

Mathematically, we can write the exponential equation of growth as follows;

[tex]\begin{gathered} P=I(1+r)^t \\ \\ \end{gathered}[/tex]

Where P is the population after a certain number of years ( 270 after 3 years

I is the initial popultaion which is 200

r is the percentage we want to calculate

t is the number of yeats to reach P which is 3 in this case

[tex]\begin{gathered} 270=200(1+r)^3 \\ (1+r)^{3\text{ }}\text{ = }\frac{270}{200} \\ \\ (1+r)^3\text{ = 1.35} \\ \\ 1\text{ + r = }\sqrt[3]{1.35} \\ \\ 1\text{ + r = 1.162} \\ \\ r\text{ = 1.162 - 1} \\ \\ r\text{ = 0.162} \end{gathered}[/tex]

To the nearest tenth of a percentage, this is 16.2%

You go on a hayride to take photographs of landmarks. The map shows your path and two landmarks. Each unit in the coordinate plane corresponds to 10 yards. Approximate your minimum distance from the giant pumpkin. If necessary, round your answer to the nearest tenth.

Answers

Using the distance between a point and a line and the conversion of the units to yards, it is found that the minimum distance from the giant pumpkin is of  8.9 yards.

What is the distance between a points and a line?

Suppose that we have a linear function defined according to the following rule, in standard notation:

Ax + By + C = 0.

And a point with coordinates given by:

P(x*,y*)

The shortest distance between the line and the point is given by:

[tex]d = \frac{|Ax^\ast + By^\ast + C|}{\sqrt{A^2 + B^2}}[/tex]

The line of the path in this problem has:

Intercept of 0, as when x = 0, y = 0.Slope of 0.5, as when x increases by 4, y increases by 2.

Hence:

y = 0.5x.

-0.5x + y = 0

-x + 2y = 0.

The coefficients are:

A = -1, B = 2.

The giant pumpkin has coordinates given by:

(x*, y*) = (-4, -3).

Hence the distance in units is given by:

d = |(-1)(-4) + 2(-3)|/sqrt(5)

d = 2/sqrt(5)

d = 0.89 units.

Each unit is equivalent to 10 yards, hence the shortest distance in yards is given by:

0.89 x 10 = 8.9 yards.

What is the missing information?

The problem is given by the image at the end of the answer.

More can be learned about the distance between two points at https://brainly.com/question/18345417

#SPJ1

Other Questions
Experts help me i will mark the brainliestIf you were inside a "time machine" and went back to ancient times, what civilization in Mesopotamia, Egypt, India or China would you want to visit and return to? why? Calculate the frequency of wave that had a wavelength of 425 nm 3. Which of the following is true about the graph of y = -x] 2? (A) The graph is increasing when x > -2. (B) The graph is decreasing when x > -2. (C) The graph is increasing when x < 0 (D) The graph is increasing when x > 0. 2(8-2x)+2(4-2x)=18 what is x? What is slope-intercept form of m=-7,b=4 Is it possible for a monotonous,constant sound to agitate a person? Explain what the average for 68,68,84,73,100,91 the mita in the inca empire was select one: a. the system of conscription labor from villages. b. the system for drafting soldiers. c. the compulsory educational system. d. the system of maintenance for the royal mummies. in. the system of collecting taxes. Kali just started a new sales floor job to save for college. She earns 15.75 plus a flat fee of 50 . She wants to earn between 200 and 400 . The following inequality represents her earning potential 200 15.75x + 50 400 Solve the inequality PLEASE HELP ASAP!! If electrons jump from a lower energy shell to a higher energy shell, they are said to be make G the subject of the formula F = GMM/1HENCE WRITE THE DIMENSION FOR G gananath obeyesekere, a sri lankan anthropologist, challenged the research findings of marshall sahlins, who argued that captain cook was killed by native hawaiians who thought he was the god lono. obeyesekeres critique raises the issue of Which region had the highest percent of export that were Articultural related to farming Companies focusing on innovation relating to product enhancement and experiential factors are all associated with what innovation zone?. Write the missing power of 10 in each equation a. 0.06 x ____=6 b. ___X 0.3 = 300 What is the equation of the line that passes through the point (-2, -2) and hasslope of 3? Find the volume of the cone. Round your answer to the nearest tenth. Use 3.14 for it.13 cm16 cmThe volume is aboutcm3 please do complete and be quick prokaryotes divide by binary fission, a form of asexual reproduction in which? Jordan and his friends bought a few items from a store. a bag of chips is 1/4 the cost of a sandwich . A soda is 3$less than a sandwich.Write an expression to represent the items